\documentclass[a4paper,10pt, table]{/media/documents/Cours/Prof/Enseignements/2014-2015/tools/style/classDS} \usepackage{/media/documents/Cours/Prof/Enseignements/2014-2015/2014_2015} \usepackage{tkz-tab} \usepackage{tkz-fct} % Title Page \titre{DS 4} % \seconde \premiereS \PSTMG \TSTMG \classe{\seconde} \date{1é décembre 2014} %\duree{1 heure} %\sujet{%{{infos.subj%}}} % DS DSCorr DM DMCorr Corr \typedoc{DS} \printanswers \begin{document} \maketitle Le barème est donné à titre indicatif, il pourra être modifié. \begin{questions} \vfill \question[5] \begin{parts} \part Quel est le nom de la figure suivante. Quelles sont les faces visibles? \begin{center} \includegraphics[scale=0.2]{./fig/pyramid.png} \end{center} \begin{solution} La figure est un pyramide. Les faces visibles sont les faces $BFC$ et $CFD$. \end{solution} \part Vous répondrez aux questions suivantes à partir du pavé droit ci-dessous. \begin{minipage}{0.4\textwidth} \includegraphics[scale=0.3]{./fig/paveDroit} \end{minipage} \begin{minipage}{0.6\textwidth} \begin{subparts} \subpart Dessiner ce pavé droit de façon à ce que les faces $AEHD$, $EHCF$ et $DHGC$ soient visibles. \begin{solution} \includegraphics[scale=0.2]{./fig/paveDroit_corr} \end{solution} \subpart Calculer le volume de ce pavé droit. \begin{solution} Volume du pavé \begin{eqnarray*} V & = & 5\times3\times4 = 60 \end{eqnarray*} \end{solution} \subpart Calculer la longueur $AC$. \begin{solution} Calcul de la longueur $AC$: Comme $ABC$ est un triangle rectangle en $B$ d'après le théorème de Pythagore, on a \begin{eqnarray*} AC^2 & = & AB^2 + BC^2\\ AC^2 &=& 5^2 + 3^2\\ AC^2 &=& 25 + 9 \\ AC^2 &=& 34\\ AC &=& \sqrt{34} \approx 5,8 \end{eqnarray*} $AC$ mesure environ 5,8cm. \end{solution} \end{subparts} \end{minipage} \end{parts} \vfill \question[5] \begin{center} \begin{tikzpicture}[scale=0.5] \repere{-6}{6}{-6}{6} \draw[very thick, color=red] plot [smooth,tension=0.5, mark=*] coordinates{(-4, -4) (-3.5, -3) (-3, 0) (-2, 1) (-1, 0) (0, -3) (1, 0) (2, -3) (2.5,0) (3, 2) (4, 3)}; \draw (4,3) node[above right] {$\mathcal{C}_f$}; \end{tikzpicture} \end{center} \begin{parts} \part Tracer le tableau de variation de le fonction représentée sur le graphique. \begin{solution} \begin{tikzpicture} \tkzTabInit[espcl=2]{$x$/1,$f(x)$/1}{-4, -2, 0, 1, 2, 4} \tkzTabVar{-/{-4}, +/{1}, -/{-3}, +/{0}, -/{-3}, +/{3}} \end{tikzpicture} \end{solution} \part Sur quels intervalles cette fonction est-elle décroissantes? \begin{solution} La fonction $f$ est décroissante sur l'intervalle $\intFF{-2}{0}$ et $\intFF{1}{2}$. \end{solution} \part Quel est le minimum de la fonction sur l'intervalle $\intFF{-1}{3}$? \begin{solution} Sur l'intevalle $\intFF{-1}{3}$, le minimum de $f$ est -3, il est atteind pour $x = 0$ ou $x = 2$ \end{solution} \part À partir de ce graphique, résoudre l'équation $f(x) = -3$ \begin{solution} Les solutions de l'équation $f(x) = -3$ sont $x = -3,5$, $x = 0$ et $x = 2$. \end{solution} \end{parts} \vfill \question[5] \begin{tikzpicture} \tkzTabInit[]{$x$/1,$g(x)$/1}{$-3$, $-0.5$, $0$,$-1$,$+\infty$} \tkzTabVar{+/{2}, -/{-3}, +/{0}, -/{1}, +/} \end{tikzpicture} \begin{parts} \part Quel est l'intervalle de définition de la fonction $g$? \begin{solution} L'intervalle de définition de $g$ est $\intFF{-3}{+\infty}$. \end{solution} \part Sur quels intervalles cette fonction est-elle décroissantes? \begin{solution} La fonction $g$ est décroissante sur les intervalles $\intFF{-3}{-0,5}$ et $\intFF{0}{1}$. \end{solution} \part Quels est le minimum de cette fonction sur l'intervalle $\intFF{-3}{1}$? \begin{solution} Sur l'intervalle $\intFF{-3}{1}$ le minimum de $g$ est -3, il est atteind pour $x=-0,5$. \end{solution} \part Tracer une fonction qui a ce tableau de variation. \begin{solution} Voici une fonction possible. \begin{center} \begin{tikzpicture}[scale=0.7] \repere{-4}{4}{-4}{5} \draw[very thick, color=red] plot [smooth,tension=0.2, mark=*] coordinates{(-3, 2) (-0.5, -3) (0, 0) (1, -1) (4, 5) }; \draw (4,5) node[above right] {$\mathcal{C}_g$}; \end{tikzpicture} \end{center} \end{solution} \part Quel est le plus grand de ces deux nombres $g(-2)$ et $g(-1)$? \begin{solution} Comme la fonction est décroissante sur $\intFF{-3}{-0,5}$ et que -2 et -1 sont dans cet intervalle. \begin{eqnarray*} -2 < -1 & \mbox{ implique } & f(-2) > f(-1) \end{eqnarray*} Donc $f(-2)$ est plus grand que $f(-1)$. \end{solution} \end{parts} \vfill \question[5] L'entreprise Cducosto produit des outils de bricolages. \begin{parts} \part Leur premier produit est un marteau. Voici le graphique représentant les bénéfices en fonction du nombre de marteau qu'elle produit et vend. \begin{center} \begin{tikzpicture}[scale=0.7] \tkzInit[xmin=0,xmax=150, ymin=-200,ymax=300, xstep=10,ystep=50] \tkzAxeX[thick, poslabel=right,label=] \tkzAxeY[thick, poslabel=above,label=] \tkzDrawX[label={\textit{Nombre de marteau}},below= -12pt] \tkzDrawY[label={\textit{Bénéfices}}, below=-10pt] \tkzGrid \tkzFct[domain=0:150,color=blue, very thick]{-0.05*\x*\x + 7.5*\x - 180} \end{tikzpicture} \end{center} \begin{subparts} \subpart Tracer le tableau de signe de cette fonction. \begin{solution} \begin{tikzpicture} \tkzTabInit[]{$x$/1,$g(x)$/1}{0, 30, 120, 150} \tkzTabLine{, -, z, +, z, -,} \end{tikzpicture} \end{solution} \subpart Sur quel intervalle doit-elle restreindre sa production pour que ses bénéfices soient positifs? \begin{solution} Pour que les bénéfices soient positifs , il faut que la production reste sur l'intervalle $\intFF{3}{120}$ \end{solution} \end{subparts} \part Leur deuxième produit est une visseuse automatique. Le bénéfice liés à cet outil est donné par la fonction suivante: \begin{eqnarray*} f:x & \mapsto & 2x - 3 \end{eqnarray*} \begin{subparts} \subpart Tracer le tableau de signe de cette fonction. \begin{solution} On cherche là où la fonction $f$ est positive \begin{eqnarray*} f(x) & > &0\\ 2x - 3 & > & 0 \\ 2x & > & 3 \\ && \mbox{2 est positif, on ne change}\\ && \mbox{le sens de l'inégalité}\\ x &>& \frac{3}{2} = 1,5 \end{eqnarray*} On ne déduit le tableau de signe (on commence le tableau en 0 car on ne peut pas produire un nombre négatif de visseuses) \begin{tikzpicture} \tkzTabInit[]{$x$/1,$f(x)$/1}{0, {1,5}, $+\infty$} \tkzTabLine{ ,-, z, +,} \end{tikzpicture} \end{solution} \subpart À partir de combien de visseuses l'entreprise fait-elle du bénéfice? \begin{solution} À partir de 2 visseuses l'entreprise fait des bénéfices (là où dans le tableau au dessus il y a un +) \end{solution} \subpart Tracer le tableau de variation de cette fonction. \begin{solution} $f$ est une fonction affine et $a = 2$ est positif. Donc $f$ est une fonction croissante \begin{tikzpicture} \tkzTabInit[espcl=2]{$x$/1,$f(x)$/1}{0, $+\infty$} \tkzTabVar{-/{}, +/{}} \end{tikzpicture} \end{solution} \end{subparts} \end{parts} \end{questions} \vfill \end{document} %%% Local Variables: %%% mode: latex %%% TeX-master: "master" %%% End: